2023 AMC 8 Problems/Problem 6

Revision as of 19:32, 24 January 2023 by Apex304 (talk | contribs)

The maximum possible value of using the digit $2,0,2,3$ is going to be $\boxed{\text{(C)}9}$ because we can maximize our value by keeping the $3$ and $2$ together in one power.